You are on page 1of 22

CPA REVIEW SCHOOL OF THE PHILIPPINES 9. Which of the following is true of the report based on agreed-upon-procedures?

Manila a. The report is restricted to those parties who have agreed to the procedures to be
AUDITING THEORY performed.
Overview of Auditing b. The CPA provides the recipients of the report limited assurance as to reasonableness of
Related PSAs : PSA 100, 120, 200 and 610 the assertion(s) presented in the financial information.
1. Certain fundamental beliefs called "postulates" underlie auditing theory. Which of the c. The report states that the auditor has not recognized any basis that requires revision of
following is not a postulate of auditing? financial statements.
a. No long-term conflict exists between the auditor and the management of the enterprise d. The report should state that the procedures performed are limited to analytical procedures
under audit. and inquiry.
b. Economic assertions can be verified. 10. Which of the following is an objective of a review engagement?
c. The auditor acts exclusively as an auditor. a. Expressing a positive opinion that the financial information is presented in conformity with
d. An audit has a benefit only to the owners. generally accepted accounting principles.
2. In all cases, audit reports must b. Expressing a limited assurance to users who have agreed as to procedures that will be
a. Be signed by the individual who performed the audit procedures. performed by the CPA.
b. Certify the accuracy of the quantitative information which was audited. c. Reporting whether material modifications should be made to such financial statements to
c. Communicate the auditors finding to the general public. make them conform with generally accepted accounting principles.
d. Inform readers of the degree of correspondence between the quantifiable information and d. Reporting that the financial statements, in all materials respects, fairly present the
the established criteria. financial position and operating results of the client.
3. The auditor communicates the results of his or her work through the medium of the 11. According to Philippine Standard on Auditing, the procedures employed in doing compilation
a. Engagement letter c. Management letter. are:
b. Audit report d. Financial statements. a. Designed to enable the accountant to express a limited assurance.
4. As used in auditing, which of the following statements best describes "assertions"? b. Designed to enable the accountant to express a negative assurance.
a. Assertions are the representations of management as to the reliability of the information c. Not designed to enable the accountant to express any form of assurance.
system. d. Less extensive than review procedures but more extensive than agreed-upon procedures.
b. Assertions are the auditor's findings to be communicated in the audit report. 12. Any services in which the CPA firm issues a written communication that express a
c. Assertions are the representations of management as to the fairness of the financial conclusion
statements. with respect to the reliability of a written assertion that is the responsibility of another party is
d. Assertions are found only in the footnotes to the financial statements. a (n)
5. The expertise that distinguishes auditors from accountants is in the a. Accounting and bookkeeping service c. Attestation service
a. Ability to interpret generally accepted accounting principles. b. Management advisory service d. Tax service
b. Requirement to possess education beyond the Bachelors degree. 13. The three types of attestation services are:
c. Accumulation and interpretation of evidence. a. Audits, review, and compilations
d. Ability to interpret ASC Statements. b. Audits, compilations, and other attestation services
6. The framework for auditing and related services as addressed by PSA excludes c. Reviews, compilations, and other attestation services
a. Review c. Compilation d. Audits, reviews, and other attestation services
b. Tax services d. Agreed upon procedure 14. Which of the following is not primary category of attestation report?
7. It refers to the level of auditors satisfaction as to the reliability of an assertion being made by a. Compilation report
one party for use by another party. b. Review report
a. Confidence level c. Assurance level c. Audit report
b. Reasonableness level d. Tolerable level d. Special audit report based on a basis of accounting other than generally accepted
8. Indicate the level of assurance provided by audit and related services. accounting principles.
abcd 15. The primary goal of the CPA in performing the attest function is to
Audit High High Negative Absolute a. Detect fraud
Review Moderate None Moderate High b. Examine individual transactions so that the auditor may certify as to their validity
Agreed-upon procedures None None None Limited c. Determine whether the client's assertions are fairly stated
Compilation None None None None d. Assure the consistent application of correct accounting procedures
16. Which of the following criteria is unique to the independent auditors attest function? d. Determine whether the audit committee of the board of directors is effectively discharging
a. General competence its responsibility to oversee managements operations
b. Familiarity with the particular industry of each client 23. A review of any part of an organizations procedures and methods for the purpose of
c. Due professional care evaluating efficiency and effectiveness is classified as a (n)
d. Independence a. Audit of financial statements c. Operational audit
17. Assurance engagement b. Compliance audit d. Production audit
a. Is an engagement in which a practitioner is engaged to issue, or does issue, a written 24. Which one of the following is more difficult to evaluate objectively?
communication that expresses a conclusion about the reliability of a written assertion that a. Efficiency and effectiveness of operations.
is the responsibility of another party. b. Compliance with government regulations.
b. Is a systematic process of objectively obtaining and evaluating evidence regarding c. Presentation of financial statements in accordance with generally accepted accounting
assertions about economic actions and events to ascertain the degree of correspondence principles.
between those assertions and established criteria and communicating the results to d. All three of the above are equally difficult..
interested users. 25. Independent auditing can best be described as a
c. Is an engagement in which the auditor provides a moderate level of assurance that the a. Branch of accounting
information subject to the engagement is free of material misstatement. b. Discipline that attests to the results of accounting and other operations and data
d. Is an engagement intended to enhance the credibility of information about a subject c. Professional activity that measures and communicates financial and business data
matter by evaluating whether the subject matter conforms in all material respects with d. Regulatory function that prevents the issuance of improper financial information
suitable criteria, thereby improving the likelihood that the information will meet the needs 26. A financial statement audit:
of an intended user. a. Confirms that financial statement assertion are accurate.
18. The single feature that most clearly distinguishes auditing, attestation, and assurance is b. Lends credibility to the financial statements.
a. Type of service. c. Scope of services. c. Guarantees that financial statements are presented fairly.
b. Training required to perform the service d. CPAs approach to the service d. Assures that fraud had been detected.
19. Identify the following as financial audit (FA), compliance audit (CA), and operational audit 27. Which of the following best describes the objective of an audit of financial statements?
(OA). a. To express an opinion whether the financial statements are prepared in accordance with
A supervisor is not carrying out his assigned responsibilities. prescribed criteria.
A companys tax return does not conform to income tax laws and regulations. b. To express an assurance as to the future viability of the entity whose financial statements
A municipalitys financial statements correctly show actual cash receipts and are being audited.
disbursements. c. To express an assurance about the managements efficiency or effectiveness in
A companys receiving department is inefficient. conducting the operations of entity.
a. CA, CA, FA, OA c. OA, CA, FA, OA d. To express an opinion whether the financial statements are prepared, in all material
b. OA, CA, CA, OA d. CA, CA, FA, CA respect, in accordance with an identified financial reporting framework.
20. The criteria for evaluating quantitative information vary. For example, in the audit of 28. Because an external auditor is paid a fee by a client company, he or she
historical a. Is absolutely independent and may conduct an audit
financial statements by CPA firms, the criteria are usually b. May be sufficiently independent to conduct an audit
a. Generally accepted auditing standards. c. Is never considered to be independent
b. Generally accepted accounting principles. d. Must receive approval of the Securities and Exchange Commission before conducting an
c. Regulations of the Internal Revenue Service. audit
d. Regulations of the Securities and Exchange Commission. 29. Which of the following is responsible for an entitys financial statements?
21. Which of the following types of audit uses as its criteria laws and regulations? a. The entitys management c. The entitys audit committee
a. Operational audit c. Financial statement audit b. The entitys internal auditors d. The entitys board of directors
b. Compliance audit d. Financial audit 30. The best statement of the responsibility of the auditor with respect to audited financial
22. An operational audit is designed to statement is:
a. Assess the efficiency and effectiveness of managements operating procedures a. The audit of the financial statements relieves management of its responsibilities
b. Assess the presentation of managements financial statements in accordance with b. The auditors responsibility is confined to his expression of opinion about the audited
generally accepted accounting principles financial statements.
c. Determine whether management has complied with applicable laws and regulations c. The responsibility over the financial statements rests with the management and the
auditor assumes responsibility with respect to the notes of financial statements. b. The entity
d. The auditor is responsible only to his unqualified opinion but not for any other type of c. The employer-employee relationship which exists for other employees in the organization
opinion. d. All of the above
31. Which of the following least likely limits the auditors ability to detect material misstatement? 39. Internal auditors cannot be independent
a. Most audit evidences are conclusive rather than being persuasive. a. Since they do not possess the CPA license.
b. The inherent limitations of any accounting and internal control system. b. Because they dont audit financial statements.
c. Audit is based on testing c. Unless their immediate supervisor is a CPA.
d. Audit procedures that are effective in detecting ordinary misstatements are ineffective in d. As long as an employer-employee relationship exists.
detecting intentional misstatements. 40. To provide for the greatest degree of independence in performing internal auditing functions,
32. Because an examination in accordance with generally accepted auditing standards is an internal auditor most likely should report to
influenced by the possibility of material errors, the auditor should conduct the examination a. Board of Directors. c. Corporate Controller.
with an attitude of b. Vice-President for Finance. d. Corporate Stockholders.
a. Professional responsiveness c. Objective judgment 41. Which statement is correct regarding the relationship between internal auditing and the
b. Conservative advocacy d. Professional skepticism external auditor?
33. Which of the following best describes why an independent auditor reports on financial a. Some judgments relating to the audit of the financial statements are those of the internal
statements? auditor.
a. Independent auditors are likely to detect fraud b. The external audit function's objectives vary according to management's requirements.
b. Competing interests may exist between management and the users of the statements c. Certain aspects of internal auditing may be useful in determining the nature, timing and
c. Misstated account balances are generally corrected by an independent audit. extent of external audit procedures.
d. Ineffective internal controls may exist. d. The external auditor is responsible for the audit opinion expressed, however that
34. An audit can have a significant effect on responsibility may be reduced by any use made of internal auditing.
a. Information Risk c. Business Risk 42. Which of the following statements is not a distinction between independent auditing and
b. The risk-free interest rate d. All of these internal auditing?
35. The main way(s) to reduce information risk is to have a. Independent auditors represent third party users external to the auditee entity, whereas
a. The user verify the information internal auditors report directly to management.
b. The user share the information risk with management b. Although independent auditors strive for both validity and relevance of evidence, internal
c. Audited financial statements provided auditors are concerned almost exclusively with validity.
d. All of the above c. Internal auditors are employees of the auditee, whereas independent auditors are
36. Which of the following is an appraisal activity established within an entity as a service to the independent contractors.
entity? d. The internal auditor's span of coverage goes beyond financial auditing to encompass
a. External auditing c. Financial auditing operational and performance auditing.
b. Internal auditing d. Compliance auditing 43. Which of the following is a correct qualification of the Chairman and Two Commissioners of
37. The scope and objectives of internal auditing vary widely and depend on the size and the Commission on Audit?
structure of the entity and the requirements of its management. Ordinarily, internal auditing a. A citizen of the Philippines.
activities include one or more of the following: b. At least 40 years of age upon appointment.
abcd c. CPAs with no less than 5 years of auditing experience or members of Philippine bar who
Review of the accounting and internal control have been engaged in law practice for at least 5 years.
systems Yes Yes Yes Yes d. Must not have been candidates for any elective position preceding appointment.
Examination of financial and operating 44. The 1986 Constitution provides that the Chairman and Commissioners of the Commission
information Yes Yes Yes No on
Review of the economy, efficiency and Audit shall be
effectiveness of operations Yes Yes No No a. All Certified Public Accountants
Review of compliance with laws, regulations b. All lawyers
and other external requirements Yes No No No c. One or two lawyers and one or two CPAs for a total of three
38. To operate effectively, an internal auditor must be independent of d. Two lawyers and one CPA
a. The line functions of the organizations 45. Which of the following is not one of the duties of the Commission on Audit
a. Define the scope of its audit and examination requirements.
b. Assume fiscal responsibility for the government and its instrumentalities 5. Users of financial statements demand independent audit because
c. Keep the general accounts of the government a. Users demand assurance that fraud does not exist
d. Promulgate accounting rules and regulations b. Management may not be objective in reporting.
46. A governmental audit may extend beyond an examination leading to the expression of an c. Users expect auditors to correct management errors.
opinion on the fairness of financial presentation to include d. Management relies on the auditor to improve internal control.
Program results Compliance Economy and efficiency 6. Which of the following types of audits is performed to determine whether an entitys financial
a. Yes Yes No statements are fairly stated in conformity with generally accepted accounting principles?
b. Yes Yes Yes a. Operational audit c. Financial statement audit
c. No Yes Yes b. Compliance audit d. Performance audit
d. No No Yes 7. Which of the following types of auditing is performed most commonly by CPAs on a
47. An audit designed to determine the extent to which the desired results of an activity contractual basis?
established by the legislative or other authorizing body are being achieved is a (an) a. Internal auditing c. Government auditing
a. Economy audit c. Program audit b. BIR auditing d. External auditing
b. Efficiency audit d. Financial related audit PSA 100 Assurance engagements
48. A government auditor evaluates a disbursement to determine if it is necessary, excessive or 8. Which of the following is incorrect regarding the Philippine Standards on Assurance
extravagant in accordance with existing rules and regulations. What kind of audit is he Engagements (PSAE)?
conducting? a. It provides an overall framework for assurance engagements intended to provide either a
Compliance audit Economy audit high or moderate level of assurance.
a. Yes No b. It provides basic principles and essential procedures for engagements intended to provide
b. No Yes a moderate level of assurance.
c. Yes Yes c. When a professional accountant is engaged to perform an assurance engagement for
d. No No which specific standards exist, those standards apply.
QUIZZERS d. If no specific standards exist for an assurance engagement, PSAE apply.
1. Which of the following is an incorrect phrase? 9. An assurance engagement should exhibit the following elements except
a. Auditing is a systematic process. a. A three party relationship c. Appropriate professional fee
b. Auditing subjectively obtains and evaluates evidence. b. A conclusion d. A subject matter
c. Auditing evaluates evidence regarding assertions. 10. Which of the following is incorrect regarding the three-party relationship element of
d. Auditing communicates results to interested users. assurance engagements?
2. Which of the following is a correct statement relating to the theoretical framework of auditing? a. Professional accountants as those persons who are members of an IFAC member body,
a. The financial data to be audited can be verified. which should be in public practice.
b. Short-term conflicts do not exist between managers who prepare data and auditors who b. The responsible party and the intended user will often be from separate organizations but
examine data. need not be.
c. Auditors do not necessarily need independence. c. The responsible party is the person or persons, either as individuals or representatives of
d. An audit has a benefit only to the owners. an entity, responsible for the subject matter.
3. The essence of the attest function is to d. The intended user is the person or class of persons for whom the professional accountant
a. Detect fraud prepares the report for a specific use or purpose.
b. Examine individual transactions so that the auditor can certify as to their validity 11. The following are assurance engagements except
c. Determine whether the clients financial statements are fairly stated a. Financial statements audit c. Review of financial statements
d. Ensure the consistent application of correct accounting procedures b. Information system reliability services d. Tax consulting
4. In auditing accounting data, the concern is with 12. Engagements frequently performed by professional accountants that are not assurance
a. Determining whether recorded information properly reflects the economic events that engagements include the following except
occurred during the accounting period. a. Agreed-upon procedures. c. Compilation
b. Determining if fraud has occurred. b. Compliance audit d. Management consulting.
c. Determining if taxable income has been calculated correctly. 13. The subject matter of an assurance engagement may take many forms, including
d. Analyzing the financial information to be sure that it complies with government a. Data b. Systems and processes c. Behavior d. All of these
14. The decision as to whether the criteria are suitable involves considering whether the subject professional connection Yes Yes No No
matter is capable of reasonably consistent evaluation against or measurement using such PSA 200 Objective and general principles governing an audit of FS
criteria. The characteristics for determining whether criteria are suitable include the following, 22. The auditors opinion
except a. Enhances the credibility of the financial statements.
a. Relevance b. Reliability: c. Understandability: d. Sufficiency b. Is an assurance as to the future viability of the entity.
15. When the professional accountant has obtained sufficient appropriate evidence to conclude c. Is an assurance as to the efficiency with which management has conducted the affairs of
that the subject matter conforms in all material respects with identified suitable criteria, he or the entity, but not effectiveness.
she can provide what level of assurance? d. Certifies the correctness of the financial statements.
a. None b. High c. Moderate d. Absolute 23. Which of the following is incorrect regarding the general principles of an audit?
16. Absolute assurance is generally not attainable as a result of such factors as: a. The auditor should comply with the Code of Ethics for Professional Ethics for Certified
abcd Public Accountants promulgated by the Philippine Professional Regulation Commission.
the use of selective testing, Yes Yes Yes No b. The auditor should conduct an audit in accordance with PSAs.
the inherent limitations of control systems Yes Yes Yes Yes c. The auditor should plan and perform an audit with an attitude of professional skepticism
the fact that much of the evidence available to recognizing that circumstances may exist that cause the financial statements to be
the professional accountant is persuasive materially misstated.
rather than conclusive Yes Yes No Yes d. The auditor would ordinarily expect to find evidence to support management
the use of judgment in gathering evidence and representations and assume they are necessarily correct.
drawing conclusions based on that evidence Yes No No No 24. It refers to the audit procedures deemed necessary in the circumstances to achieve the
PSA 120 Framework of PSA objective of the audit.
17. The Framework of PSA applies to a. Scope of an audit c. Objective of an audit
a. Taxation b. Consultancy c. Accounting advice d. Compilation b. Audit program d. Reasonable assurance
18. Agreed-upon procedures provides what level of assurance? 25. Which of the following are sources of procedures to be considered by the auditor to conduct
a. None b. High c. Moderate d. Absolute an audit in accordance with PSAs?
19. Which of the following procedures ordinarily performed during an audit are also performed in PSA Legislation Terms of Audit Engagement Type of Opinion
review? a. Yes No No No
a. Assessment of accounting and internal control systems b. No No Yes Yes
b. Test of controls c. No Yes Yes No
c. Tests of records and of responses to inquiries d. Yes Yes Yes No
d. Inquiry and analytical procedures
20. The objective of a review of financial statements
CPA REVIEW SCHOOL OF THE PHILIPPINES
Manila
a. Is to enable the auditor to express an opinion whether the financial statements are
prepared, in all material respects, in accordance with an identified financial reporting AUDITING THEORY
framework. PRE-ENGAGEMENT
b. Is to enable an auditor to state whether, on the basis of procedures which do not provide Related PSAs: PSA 210
all the evidence that would be required in an audit, anything has come to the auditors PSA 210 - Terms of Audit Engagements
attention that causes the auditor to believe that the financial statements are not prepared, The auditor and the client should agree on the terms of the engagement. The agreed terms
in all material respects, in accordance with an identified financial reporting framework. would
c. Is to carry out those procedures of an audit nature to which the auditor and the entity and need to be recorded in an audit engagement letter or other suitable form of contract.
any appropriate third parties have agreed and to report on factual findings. It is in the interest of both client and auditor that the auditor sends an engagement letter,
d. Is to use accounting expertise as opposed to auditing expertise to collect, classify and preferably
summarize financial information. before the commencement of the engagement, to help in avoiding misunderstandings with
21. An auditor is associated with financial information when respect
abcd to the engagement.
the auditor attaches a report to that The engagement letter documents and confirms:
information Yes No Yes No 1. the auditors acceptance of the appointment;
consents to the use of the auditors name in a 2. the objective and scope of the audit;
3. the extent of the auditors responsibilities to the client; and the engagement.
4. the form of any reports. The auditor may decide not to send a new engagement letter each period. However, the
Principal Contents following
The form and content of audit engagement letters may vary for each client, but they would factors may make it appropriate to send a new letter:
generally include reference to: Any indication that the client misunderstands the objective and scope of the audit.
The objective of the audit of financial statements. Any revised or special terms of the engagement.
Managements responsibility for the financial statements. A recent change of senior management, board of directors or ownership.
The scope of the audit, including reference to applicable legislation, regulations, or A significant change in nature or size of the clients business.
pronouncements of professional bodies to which the auditor adheres. Legal requirements.
The form of any reports or other communication of results of the engagement. Acceptance of a Change in Engagement
The fact that because of the test nature and other inherent limitations of an audit, together A request from the client for the auditor to change the engagement may result from:
with the inherent limitations of any accounting and internal control system, there is an 1. a change in circumstances affecting the need for the service;
unavoidable risk that even some material misstatement may remain undiscovered. 2. a misunderstanding as to the nature of an audit or related service originally requested; or
Unrestricted access to whatever records, documentation and other information requested 3. a restriction on the scope of the engagement, whether imposed by management or caused
in connection with the audit. by circumstances.
The auditor may also wish to include in the letter: Items 1 and 2 would ordinarily be considered a reasonable basis for requesting a change in the
Arrangements regarding the planning of the audit. engagement. In contrast a change would not be considered reasonable if it appeared that the
Expectation of receiving from management written confirmation concerning change relates to information that is incorrect, incomplete or otherwise unsatisfactory.
representations made in connection with the audit. If the auditor agreed to a change of the engagement:
Request for the client to confirm the terms of the engagement by acknowledging receipt the auditor and the client should agree on the new terms;
of the engagement letter. the report issued would be that appropriate for the revised terms of engagement; and
Description of any other letters or reports the auditor expects to issue to the client. in order to avoid confusing the reader, the report would not include reference to:
Basis on which fees are computed and any billing arrangements. (a) The original engagement; or
When relevant, the following points could also be made: (b) Any procedures that may have been performed in the original engagement, except
Arrangements concerning the involvement of other auditors and experts in some aspects where the engagement is changed to an engagement to undertake agreed-upon
of the audit. procedures and thus reference to the procedures performed is a normal part of the
Arrangements concerning the involvement of internal auditors and other client staff. report.
Arrangements to be made with the predecessor auditor, if any, in the case of an initial If the auditor is unable to agree to a change of engagement and is not permitted to continue the
audit. original agreement:
Any restriction of the auditors liability when such possibility exists. the auditor should withdraw; and
A reference to any further agreements between the auditor and the client. consider whether there is any obligation, either contractual or otherwise, to report to other
Audits of Components parties, such as the board of directors or shareholders, the circumstances necessitating
When the auditor of a parent entity is also the auditor of its subsidiary, branch or division the withdrawal.
(component), the factors that influence the decision whether to send a separate engagement MULTIPLE CHOICE QUESTIONS
letter 1. Prior to the acceptance of an audit engagement with a client who has terminated the services
to the component include: of the predecessor auditor, the CPA should
Who appoints the auditor of the component. a. Contact the predecessor auditor without advising the prospective client and request a
Whether a separate audit report is to be issued on the component. complete report of the circumstance leading to the termination with the understanding that
Legal requirements. all information disclosed will be kept confidential.
The extent of any work performed by other auditors. b. Accept the engagement without contacting the predecessor auditor since the CPA can
Degree of ownership by parent. include audit procedures to verify the reason given by the client for the termination.
Degree of independence of the components management. c. Not communicate with the predecessor auditor because this would in effect be asking the
Recurring Audits auditor to violate the confidential relationship between auditor and client.
On recurring audits, the auditor should consider whether circumstances require the terms of the d. Advise the client of the intention to contact the predecessor auditor and request
engagement to be revised and whether there is a need to remind the client of the existing terms permission for the contact.
of 2. Before accepting an audit engagement, a successor auditor should make specific inquiries of
the predecessor auditor regarding the predecessors 9. Which of the following is appropriately included in an audit engagement letter?
a. Opinion of any subsequent events occurring since the predecessors audit report was I. Because of the test nature and other inherent limitations of an audit, together with the
issued. inherent limitations of any accounting and internal control system, there is an unavoidable
b. Understanding as to the reasons for the change of auditors. risk that even some material misstatements may remain undiscovered.
c. Awareness of the consistency in the application of GAAP between periods. II. The audit will be made with the objective of expressing an opinion on the financial
d. Evaluation of all matters of continuing accounting significance. statements.
3. A successor auditor most likely would make specific inquiries of the predecessor auditor III. An audit also includes assessing the accounting procedures used and significant
regarding estimates made by management.
a. Specialized accounting principles of the clients industry. a. I and II c. II and III
b. The competency of the clients internal audit staff. b. I and III d. I, II and III
c. The uncertainty inherent in applying sampling procedures. 10. Which of the following is least likely included in an audit engagement letter?
d. Disagreements with management as to auditing procedures. a. The objective of financial reporting.
4. Which of the following should an auditor obtain from the predecessor auditor prior to b. Management responsibility for the financial statements.
accepting an audit engagement? c. The form of any reports or other communication of the results of the engagement.
a. Analysis of balance sheet accounts d. Arrangement concerning the involvement of other auditors or experts in some aspects of
b. Analysis of income statement accounts the audit.
c. All matters of continuing accounting significance 11. An audit engagement letter least likely includes
d. Facts that might bear on the integrity of management a. A reference to the inherent limitation of an audit that some material misstatements may
5. When an independent auditor is approached to perform an audit for the first time, he or she remain undiscovered.
should make inquiries of the predecessor auditor. Inquiries are necessary because the b. Identification of specific audit procedures that the auditor needs to undertake.
predecessor may be able to provide the successor with information that will assist the c. Description of any letters or reports that the auditor expects to submit to the client.
successor in determining whether d. Arrangements concerning the involvement of internal auditors and other clients staff.
a. The predecessors work should be used. 12. Which of the following least likely requires the auditor to send a new engagement letter?
b. The company rotates auditors. a. An indication that the client misunderstands the objective and scope of the audit.
c. In the predecessors opinion, control risk is low. b. Any revised or special terms of the engagement.
d. The engagement should be accepted. c. A recent change in the audit firms management.
6. If permission from client to discuss its affairs with the proposed auditor is denied by the client, d. Legal requirements and other government agencies pronouncements.
the predecessor auditor should: 13. Which of the following least likely influence the auditors decision to send a separate
a. Keep silent of the denial. engagement letter to a component of parent entity client?
b. Disclose the fact that the permission to disclose is denied by the client. a. Legal requirements
c. Disclose adequately to proposed auditor all noncompliance made by the client. b. Degree of ownership over a component entity by parent company
d. Seek legal advice before responding to the proposed auditor c. Location of the principal place of business of the component entity
7. The objective and scope of the audit and the extent of the auditors responsibilities to the d. Who appoints the auditor of the component
client are best documented in 14. According to PSA 210, which of the following statements is correct?
a. Independent auditors report c. Clients representation letter a. The auditor and the client need not agree on the terms of the engagement.
b. Audit engagement letter d. Audit program b. Where the terms of the engagement are changed, the auditor and the client need not
8. The following are valid reasons why an auditor sends to his client an engagement letter: agree on the new terms if they already agreed on the old terms.
ABCD c. The engagement letter assists in the supervision and review of the audit work.
a. Avoid misunderstanding with respect to d. The auditor may agree to a change of engagement where there is reasonable justification
engagement for doing so.
Yes Yes No Yes 15. Which of the following is a NOT valid reason for a change of the engagement to a lower
b. Confirms the auditors acceptance of the level
appointment of assurance?
Yes Yes Yes No a. Change in circumstances affecting the need for the service.
c. Objective and scope of the audit Yes Yes Yes Yes b. Restriction on the scope of the engagement.
d. Assures CPAs compliance to GAAS Yes No No Yes c. Misunderstanding as to the nature of the engagement originally requested.
d. The clients need is satisfied by an engagement that provides lower level of assurance. b. To promulgate auditing standards, practices and procedures that shall be generally
16. When a change in the type of engagement from higher to lower level of assurance is accepted by the accounting profession in the Philippines.
reasonably justified, the report based on the revised engagement c. To assist the Board of Accountancy in conducting administrative proceedings on erring
a. Should contain a separate paragraph that refers to the original engagement. CPAs in audit practice.
b. Should always refer to any procedures that may have been performed in the original d. To undertake continuing research on both auditing and financial accounting in order to
engagement. make them responsive to the needs of the public.
c. Should qualify the opinion due to scope limitation. 6. In the absence of pronouncements issued by the ASPC and the PICPA, published
d. Omits reference to the original engagement. statements and guidelines issued by other authoritative bodies like AICPA, IAASB and AFA
17. Which of the following actions may be appropriate if the auditor is unable to agree to a are the bases of determining generally accepted auditing standards (GAAS). What effect do
change of the engagement and is not permitted to continue the original engagement these pronouncements provide in determining the GAAS?
I. Issue a qualified opinion due to a significant scope limitation. a. Authoritative b. Persuasive c. Parallel d. Alternative
II. Auditor should withdraw from the engagement. 7. Which statement is incorrect regarding the pronouncements of ASPC?
III. Consider whether there is any obligation to report to the board of directors or a. The PSAs and Interpretations may also have application, as appropriate, to other related
shareholders the circumstances necessitating withdrawal activities of auditors.
a. I only c. II and III b. PSAs contain basic principles and essential procedures (identified in bold type black
b. I and II d. I, II and III lettering) together with related guidance in the form of explanatory and other material.
c. PSAs need only be applied to material matters.
CPA REVIEW SCHOOL OF THE PHILIPPINES d. The Interpretations have the same authority as the PAPSs.
Manila 8. The Philippine Standards on Auditing issued by ASPC
AUDITING THEORY a. Apply to independent examination of financial statements of any entity when such an
Professional Accounting Practice examination is conducted for the purpose of expressing an opinion thereon.
Related PSA : Preface to PSA and Related Services b. Must not apply to other related activities of auditors
1. The following statements relate to the accounting profession: c. Need to be applied on all audit related matters.
I. To merit public trust and confidence, the professional person must convince the public that d. Require that in no circumstances would an auditor may judge it necessary to depart from
he will place public service ahead of personal reward. a PSA, even though such a departure may result to more effective achievement of the
II. A CPA certificate is evidence of basic competence in the discipline of accounting at the time objective of an audit
the certificate is granted. 9. These statements are issued to provide practical assistance to auditors in implementing the
III. A code of professional conduct is one of the most important distinguishing characteristics of PSAs
a profession. a. Interpretations b. SASP c. PAPS d. SPA
State whether the foregoing statements are true or false. 10. A body that is created through the Philippine Accountancy Act of 2004 and is intended to
a. All of the statements are true. c. Only two of the statements are true. replace the ASPC.
b. Only one of the statements is true. d. All of the statements are false. a. Auditing and Assurance Standards Council (AASC)
2. Which of the following is not normally a service rendered by public accountants? b. Financial Reporting Standards Council (FRSC)
a. Management consultation service c. Internal auditing c. Education Technical Council (ETC)
b. Attest function d. Taxation d. Philippine Institute of Certified Public Accountants (PICPA)
3. A CPA firm offers management advisory services to clients. Its primary purpose is to 11. Which of the following government agencies is represented both to the Auditing Standards
a. Furnish professional advice and assistance which will enable the client to improve and Practices Council and the Auditing and Assurance Standards Council?
operations. a. Bangko Sentral ng Pilipinas c. Securities and Exchange Commission
b. Keep the CPA firm competitive with other firms. b. Bureau of Internal Revenue d. Commission on Higher Education
c. Establish the firm as a consultant, thus ensuring its future expansion and growth. 12. Are the following government agencies represented both to Auditing Standards and
d. Permit the firms staff members to acquire expertise in other areas of practice. Practices
4. The government agency tasked by law of implementing and enforcing the regulatory policies Council (ASPC) and the new Auditing and Assurance Standards Council (AASC)?
of the national government with respect to the regulation and licensing of the various abcd
professions and occupations under its jurisdiction is Board of Accountancy Yes Yes Yes Yes
a. PRC b. BOA c. COA d. SEC Securities and Exchange Commission Yes Yes No No
5. Which of the following mostly describes the function of ASPC? Commission on Audit Yes Yes Yes Yes
a. To monitor full compliance by auditors to PSAs.
Bangko Sentral ng Pilipinas Yes No Yes No professional accounting practice. Which of them is the exception?
13. Which statement is correct regarding AASC? a. Performing audits or verification of financial transactions and records for more than one
a. The AASC shall be composed of 15 members plus a Chairman. client.
b. The chairman and members of the AASC shall be appointed by the President of the b. Employed as the department chairman that supervises the BSA program of an
Philippines upon the recommendation of PRC. educational institution.
c. The chairman and members of the AASC shall have a non-renewable term of 3 years. c. Employment as controller of a private business enterprise and such employment requires
d. The chairman should have been or presently a senior practitioner in public accountancy. that the holder thereof should be a CPA.
14. The following sectors represented by the PICPA to the membership of AASC have one d. Appointment in the government where first grade civil service eligibility is a prerequisite.
representative, except 3. A person is not deemed to be engaged in professional accounting practice if
a. Government c. Commerce and industry a. Her merely holds himself out as skilled in the science and practice of accounting and
b. Public practice d. Academe qualified to render services as a CPA.
15. Statements on financial accounting standards constituting GAAP are issued by the b. He merely offers to render services as a CPA to the public, but does not actually render
a. Philippine Institute of CPAs. c. Audit Standards and Practices Council. such services.
b. Securities and Exchange Commission. d. Accounting Standards Council. c. He offers or renders bookkeeping services to more than one client.
16. Indicate whether the following functions would be performed by: d. He installs and revises accounting systems for more than one client.
P Partner S Senior 4. Practice in Public Accountancy shall constitute in a person
M Manager AS Audit Assistant a. Involved in decision making requiring professional knowledge in the science of
(1) Supervises two or more concurrent audit engagements accounting, or when such employment or position requires that the holder thereof must be
(2) Performs detailed audit procedures a certified public accountant.
(3) Overall responsibility for audit b. In an educational institution which involve teaching of accounting, auditing, management
(4) Signs audit report advisory services, finance, business law, taxation, and other technically related subjects.
(1) (2) (3) (4) c. Who holds, or is appointed to, a position in an accounting professional group in
a. P AS S M government or in a government owned and/or controlled corporation, including those
b. M S M P performing proprietary functions, where decision making requires professional knowledge
c. M AS P P in the science of accounting,
d. P AS S M d. Holding out himself/herself as one skilled in the knowledge, science and practice of
17. The amount of audit fees depend largely on the accounting, and as a qualified person to render professional services as a certified public
a. Size and capitalization of the company under audit. accountant; or offering or rendering, or both, to more than one client on a fee basis or
b. Amount of profit for the year. otherwise.
c. Availability of cash. 5. Any position in any business or company in the private sector which requires supervising the
d. Volume of audit work and degree of competence and responsibilities involved. recording of financial transactions, preparation of financial statements, coordinating with the
18. In determining audit fees, an auditor may take into account each of the following except external auditors for the audit of such financial statements and other related functions shall
a. Volume and intricacy of work involved. c. Number and cost of manhours needed. be occupied only by a duly registered CPA. Provided (choose the incorrect one)
b. Degree of responsibility assumed. d. Size and amount of capital of client. a. That the business or company where the above position exists has a paid-up capital of at
19. Under this method of billing a client, the external auditors charges on the basis of time spent least P5,000,000 and/or an annual revenue of at least P10,000,000.
by principals/partners, supervisors, seniors and juniors at predetermined rates agreed upon b. The above provision shall apply only to persons to be employed after the effectivity of the
with the client Implementing Rules and Regulations of RA 9298.
a. Maximum fee basis c. Flat sum basis c. The above provision shall not result to deprivation of the employment of incumbents to
b. Retainer basis d. Per diem basis the position.
RA No. 9298 Philippine Accountancy Act of 2004 and its IRR d. None of the above.
1. Which of the following is not one of the specified objectives of the Accountancy Act of 2004? 6. The integrated national professional organization of Certified Public Accountants accredited
a. Examination for registration of CPAs. by the BOA and the PRC per PRC accreditation No. 15 dated October 2, 1975.
b. Supervision, control, and regulation of accounting practice. a. Auditing and Assurance Standards Council (AASC)
c. Standardization and regulation of accounting education. b. Financial Reporting Standards Council (FRSC)
d. Promulgation of accounting and auditing standards. c. Education Technical Council (ETC)
2. In all of the following situations except one, a person is deemed to be engaged in d. Philippine Institute of Certified Public Accountants (PICPA)
7. As defined in the IRR of RA 9298, it is an organization engaged in the practice of public b. Revocation of CPA certificate d. Suspension of CPA certificate
accountancy, consisting of sole proprietor, either alone or with one or more staff members. 14. The creation of FRSC and AASC is intended to assist the BOA in carrying out its function to
a. Firm b. Individual CPA c. Partnership d. Sector a. To monitor the conditions affecting the practice of accountancy and adopt such
8. The following statements relate to the Board of Accountancy. Which statement is correct? measures, rules and regulations and best practices as may be deemed proper for the
a. The Board consists of a Chairman and six members. enhancement and maintenance of high professional, ethical, accounting and auditing
b. The chairman and members are appointed by the President of the Philippines upon standards.
recommendation of PICPA. b. To supervise the registration, licensure and practice of accountancy in the Philippines.
c. The Professional Regulation Commission may remove from the Board any member c. To prescribe and adopt the rules and regulations necessary for carrying out the provisions
whose certificate to practice has been removed or suspended. of RA 9298.
d. Majority of the board members shall as much as possible be in public practice. d. To prepare, adopt, issue or amend the syllabi of the subjects for examinations.
9. The APO shall submit its nominations with complete documentation to the Commission not 15. A body that is created to assist the BOA in the attainment of the objective of continuously
later than _____ prior to the expiry of the term of an incumbent chairman or member. upgrading the accountancy education in the Philippines to make the Filipino CPAs globally
a. 30 days b. 60 days c. 90 days d. 120 days competitive.
10. A member of the BOA shall, at the time of his/her appointment, possess the following a. Philippine Institute of Certified Public Accountants (PICPA)
qualifications, except b. Education Technical Council (ETC)
a. Must be a natural-born citizen and resident of the Philippines. c. Financial Reporting Standards Council (FRSC)
b. Must be a duly registered CPA with more than ten (10) years of work experience in any d. Associations of CPAs in Education (ACPAE)
scope of practice of accountancy. 16. Which of the following is are grounds for suspension or removal of members of BOA?
c. Must be of good moral character and must not have been convicted of crimes involving I. Neglect of duty or incompetence.
moral turpitude. II. Violation or tolerance of any violation of the CPAs Code of Ethics.
d. Must not be a director or officer of the APO at the time of his/her appointment. III. Final judgment of crimes involving moral turpitude.
11. Which statement is incorrect regarding the term of office of the chairman and the members IV. Rigging of the certified public accountants licensure examination results.
of a. I, II, III and IV b. I, II and III c. III and IV d. I, III and IV
the Board of Accountancy (BOA)? 17. The following statements relate to CPA examination ratings. Which of the following is
a. The Chairman and members of the Board shall hold office for a term of three years. incorrect?
b. No person who has served two (2) successive complete terms shall be eligible for a. To pass the examination, candidates should obtain a general weighted average of 75%
reappointment until the lapse of one (1) year. and above, with no rating in any subject less than 65%.
c. A person may serve the BOA for not more than twelve years. b. Candidates who obtain a rating of 75% and above in at least four subjects shall receive a
d. A member of the BOA may continuously serve office for more than nine years. conditional credit for the subjects passed.
12. The Board shall exercise the following specific powers, functions and responsibilities: c. Candidates who failed in four complete examinations shall no longer be allowed to take
abcd the examinations the fifth time.
To supervise the registration, licensure and d. Conditioned candidates shall take an examination in the remaining subjects within two
practice of accountancy Yes Yes Yes Yes years from the preceding examination.
To issue, suspend, revoke, or reinstate the 18. The Board, subject to the approval of the Commission, may revise or exclude any of the
Certificate of Registration for the practice of subjects and their syllabi, and add new ones as the need arises. Provided that the change
the accountancy profession Yes No Yes Yes shall not be more often than every
To monitor the conditions affecting the practice a. 2 years b. 3 years c. 4 years d. 5 years
of accountancy Yes Yes No Yes 19. The BOA shall submit to the PRC the ratings obtained by each candidate within how many
To conduct an oversight into the quality of calendar days after the examination?
audits of financial statements Yes No Yes No a. 5 days b. 10 days c. 15 days d. 30 days
To issue a cease or desist order to any 20. A Professional Identification Card bearing the registration number, date of issuance, expiry
person, association, partnership or corporation date, duly signed by the chairperson of the Commission, shall be issued to every registrant
engaged in violation of any provision of the Act Yes Yes No Yes renewable every
13. Which of the following is not one of the penalties that can be imposed by the Board of a. Two years b. Three years c. Four years d. Five years
Accountancy? 21. The certified public accountant shall be required to indicate which of the following numbers
a. Fine or imprisonment c. Reprimand on the documents he/she signs, uses or issues in connection with the practice of his/her
profession? related functions.
Abcd d. Public practice and shall include at least two years as audit assistant and at least one
His/her Certificate of Registration Yes Yes Yes No year as auditor in charge of audit engagement covering full audit functions of significant
Professional Identification Card Yes Yes Yes Yes clients.
Professional Tax Receipt Yes Yes No Yes 28. The Accountancy Law provides that all working papers made during an audit shall be the
Telephone Yes No No No property of the auditor. These working papers shall include the following, except:
22. The BOA shall not refuse the registration of any person who successfully passed the CPA a. Schedules and memoranda made by the CPA and his staff.
examinations if b. Working papers prepared and submitted by the client.
a. Convicted by a court of competent jurisdiction of a criminal offense involving moral c. Excerpts or copies of documents furnished the auditor.
turpitude d. Reports submitted by the CPA to the client.
b. Convicted for a political offense. 29. Individual CPAs, Firms or Partnerships of CPAs, including partners and staff members
c. Guilty of immoral and dishonorable conduct thereof shall register with the BOA and the PRC. If the application for registration of AB and
d. None of the above. Co., CPAs was approved on August 30, 2005, it shall file for renewal on or before
23. Which of the following is not one of the grounds for proceedings against a CPA? a. September 30, 2007 c. December 31, 2007
a. Gross negligence or incompetence in the practice of his profession. b. September 30, 2008 d. August 30, 2008
b. Engaging in public practice while being employed in a private enterprise. 30. Which statement is correct regarding CPE requirements for renewal of professional license?
c. Insanity. a. The total CPE credit units required for CPAs shall be sixty (60) units for three (3) years,
d. Immoral or dishonorable conduct. provided that a minimum of twenty (20) credit units shall be earned in each year.
24. A person whose CPA certificate has been revoked b. A registered professional shall be permanently exempted from CPE requirements upon
a. Can no longer be reinstated as a CPA reaching the age of 60 years old.
b. Is automatically reinstated as a CPA after two years if the has acted in an exemplary c. A registered professional who is working abroad shall be temporarily exempted from
manner compliance with CPE requirement during his/her stay abroad, provided that he/she is has
c. May be reinstated as a CPA by the Board of Accountancy after two years if he has acted been out of the country for at least one year immediately prior to the date of renewal.
in an exemplary manner d. Those who failed to renew professional licenses for a period of five (5) continuous years
d. May be reinstated by the PRC after two years if he has acted in an exemplary manner from initial registration, or from last renewal shall be declared delinquent.
25. Who is not permitted by law to practice accountancy? 31. Any person who shall violate any of the provisions of RA 9298 or any of its implementing
a. A corporation whose stockholders are all CPAs rules and regulations as promulgated by the Board subject to the approval of the
b. A partnership of CPAs Commission, shall, upon conviction, be punished by
c. An individual CPA practitioner a. A fine of not less than fifty thousand pesos (P 50,000.00) or by imprisonment for a period
d. A partnership of CPAs with some non-CPA staff not exceeding two (2) years or both.
26. A certificate of accreditation shall be issued to certified public accountants in public practice b. A fine of not less than one hundred thousand pesos (P 100,000.00) or by imprisonment
only upon showing, in accordance with rules and regulations promulgated by the Board and for a period not exceeding two (2) years or both.
approved by the Commission, that such registrant has acquired how many years of c. A fine of not less than fifty thousand pesos (P 50,000.00) or by imprisonment for a period
meaningful experience in any of the areas of public practice? not exceeding three (3) years or both.
a. One b. Two c. Three d. Four d. A fine of not less than one hundred thousand pesos (P 100,000.00) or by imprisonment
27. A meaningful experience shall be considered as satisfactory compliance with the for a period not exceeding three (3) years or both.
requirements of Section 28 of RA 9298 if it is earned in (Choose the incorrect one) 32. The primary duty to enforce the provisions of RA 9298 and its IRR rests with
a. Commerce and industry and shall include significant involvement in general accounting, a. The PRC c. The PRC and BOA
budgeting, tax administration, internal auditing, liaison with external auditors, representing b. The BOA d. The AASC
his/her employer before government agencies on tax and matters related to accounting or 33. The PICPA shall renew its Certificate of Accreditation once every how many years after the
any other related functions. date of the Resolution granting the petition for re-accreditation and the issuance of the said
b. Academe/education and shall include teaching for at least three (3) trimesters or two (2) certificate upon submission of the requirements?
semesters subjects in either financial accounting, business law and tax, auditing a. 2 years b. 3 years c. 4 years d. 6 years
problems, auditing theory, financial management and management services. 34. Below are the names of three CPA firms and pertinent facts relative to each firm. Unless
c. Government and shall include significant involvement in general accounting, budgeting, otherwise indicated, the individuals named are CPAs and partners, and there are no other
tax administration, internal auditing, liaison with the Commission on Audit or any other partners. Which firm name and related facts indicates a violation of the IRR of RA 9298?
a. Joyce, Ara and Angela, CPAs (Joyce died about 10 years ago, Ara and Angela are I. Misstatements resulting from fraudulent financial reporting
continuing the firm) II. Misstatements resulting from misappropriation of assets
b. Lupin and Fujico, CPAs ( the name of Goymon a third active partner is omitted in the firm a. I and II b. I only c. II only d. Neither I nor II
name) 6. Fraudulent financial reporting involves intentional misstatements or omissions of amounts or
c. Hugo and Pugo, CPAs (Hugo died 25 months ago, Pugo is continuing the firm as a sole disclosures in financial statements to deceive financial statement users. Fraudulent financial
proprietor) reporting least likely involve
d. Bubu and Bibi, CPAs (Bibi died 3 years ago, Bobot was admitted into the partnership 2 a. Deception such as manipulation, falsification, or alteration of accounting records or
months after Bibis death.) supporting documents from which the financial statements are prepared.
b. Misrepresentation in, or intentional omission from, the financial statements of events,
transactions or other significant information.
CPA REVIEW SCHOOL OF THE PHILIPPINES c. Intentional misapplication of accounting principles relating to measurement, recognition,
Manila classification, presentation, or disclosure.
AUDITING THEORY d. Embezzling receipts, stealing physical or intangible assets, or causing an entity to pay for
PROFESSIONAL AND LEGAL RESPONSIBILITIES goods and services not received.
Related PSAs : PSA 240rev, 250 and 260 7. Which of the following illustrates a perceived opportunity to commit fraud?
PSA 240(rev) The Auditors Responsibility to Consider Fraud and Error in the Audit a. Individuals are living beyond their means.
of FS b. Management is under pressure, from sources outside or inside the entity, to achieve an
1. The primary responsibility for the prevention and detection of fraud and error rests with expected (and perhaps unrealistic) earnings target.
a. The auditor. c. The management of an entity. c. An individual believes internal control could be circumvented because the individual is in a
b. Those charged with governance. d. Both b and c. position of trust or has knowledge of specific weaknesses in the internal control system.
2. When planning and performing audit procedures and evaluating and reporting the results d. All of the above.
thereof, the auditor should 8. Which statement is incorrect regarding the auditors responsibility to consider fraud and error
a. Search for errors that would have a material effect and for fraud that would have either in
material or immaterial effect on the financial statements. an audit of financial statements?
b. Consider the risk of misstatements in the financial statements resulting from fraud or error. a. The auditor is not and cannot be held responsible for the prevention of fraud and error.
c. Search for fraud that would have a material effect and for errors that would have either b. In planning the audit, the auditor should discuss with other members of the audit team the
material or immaterial effect on the financial statements. susceptibility of the entity to material misstatements in the financial statements resulting from
d. Consider the risk of material misstatements in the financial statements resulting from fraud fraud or error.
or error. c. The auditor should design test of controls to reduce to an acceptably low level the risk that
3. The following are examples of error, except misstatements resulting from fraud and error that are material to the financial statements
a. A mistake in gathering or processing data from which financial statements are prepared. taken as a whole will not be detected.
b. An incorrect accounting estimate arising from oversight or misinterpretation of facts. d. When the auditor encounters circumstances that may indicate that there is a material
c. A mistake in the application of accounting principles relating to measurement, recognition, misstatement in the financial statements resulting from fraud or error, the auditor should
classification, presentation, or disclosure. perform procedures to determine whether the financial statements are materially misstated.
d. Misrepresentation in the financial statements of events, transactions or other significant 9. The risk of not detecting a material misstatement resulting from fraud is higher than the risk of
information. not detecting a material misstatement resulting from error because
4. The term fraud refers to an intentional act by one or more individuals among management, a. The effect of fraudulent act is likely omitted in the accounting records.
those charged with governance, employees, or third parties, involving the use of deception to b. Fraud is ordinarily accompanied by acts specifically designed to conceal its existence.
obtain an unjust or illegal advantage. Which statement is correct regarding fraud? c. Fraud is always a result of connivance between or among employees.
a. Auditors make legal determinations of whether fraud has actually occurred. d. The auditor is responsible to detect errors but not fraud.
b. Misstatement of the financial statements may not be the objective of some frauds. 10. Which of the following statements describes why a properly designed and executed audit
c. Fraud involving one or more members of management or those charged with governance is may
referred to as employee fraud. not detect a material fraud?
d. Fraud involving only employees of the entity is referred to as management fraud. a. Audit procedures that are effective for detecting an unintentional misstatement may be
5. The types of intentional misstatements that are relevant to the auditors consideration of fraud ineffective for an intentional misstatement that is concealed through collusion.
include b. An audit is designed to provide reasonable assurance of detecting material errors, but there
is no similar responsibility concerning material fraud. b. Factors whose presence often has been observed in circumstances where frauds have
c. The factors considered in assessing control risk indicated an increased risk of intentional occurred.
misstatements, but only a low risk of unintentional errors in the financial statements. c. Factors whose presence requires modifications of planned audit procedures.
d. The auditor did not consider factors influencing audit risk for account balances that have d. Reportable conditions identified during an audit.
pervasive effects on the financial statements taken as a whole. 17. Which of the following is least likely a category of fraud risk factors that relate to
11. The auditors ability to detect a fraud depends on factors such as misstatements
I. The skillfulness of the perpetrator. resulting from fraudulent financial reporting?
II. The frequency and extent of manipulation. a. Managements characteristics and influence over the control environment.
III. The degree of collusion involved. b. Industry conditions.
IV. The relative size of individual amounts manipulated. c. Operating characteristics and financial stability.
V. The seniority of those involved. d. Susceptibility of assets to misappropriation.
a. All of the above b. I, III and V only c. I, II, III and V only d. III and V only 18. Fraud risk factors relating to managements characteristics and influence over the control
12. In comparing management fraud with employee fraud, the auditors risk of failing to discover environment
the a. Pertain to managements abilities, pressures, style, and attitude relating to internal control
fraud is and the financial reporting process.
a. Greater for employee fraud because of the higher crime rate among blue collar workers. b. Involve the economic and regulatory environment in which the entity operates.
b. Greater for management fraud because of managements ability to override existing internal c. Pertain to the nature and complexity of the entity and its transactions, the entitys financial
controls. condition, and its profitability.
c. Greater for employee fraud because of the larger number of employees in the organization. d. Involve the lack of controls designed to prevent or detect misappropriation of assets.
d. Greater for management fraud because managers are inherently smarter than employees. 19. Which of the following is least likely an example of fraud risk factors relating to
13. The subsequent discovery of a material misstatement of the financial statements resulting managements
from characteristics and influence over the control environment?
fraud or error, in and of itself, indicates: a. There is motivation for management to engage in fraudulent financial reporting.
abcd b. There is a failure by management to display and communicate an appropriate attitude
a failure to obtain reasonable assurance Yes Yes Yes No regarding internal control and the financial reporting process.
inadequate planning, performance or judgment Yes No No No c. Non-financial management participates excessively in, or is preoccupied with, the selection
the absence of professional competence and of accounting principles or the determination of significant estimates.
due care Yes Yes No No d. New accounting, statutory or regulatory requirements that could impair the financial stability
a failure to comply with PSAs Yes No No No or profitability of the entity.
14. Whether the auditor has performed an audit in accordance with PSAs is determined by 20. The following are examples of fraud risk factors relating to industry conditions, except
a. The adequacy of the audit procedures performed in the circumstances and the suitability of a. There is a high turnover of management, counsel or board members.
the auditors report based on the result of these procedures. b. A high degree of competition or market saturation, accompanied by declining margins.
b. The absence of material misstatements. c. A declining industry with increasing business failures and significant declines in customer
c. The absence of material errors. demand.
d. The Securities and Exchange Commission. d. Rapid changes in the industry, such as high vulnerability to rapidly changing technology or
15. When planning the audit, which of the following is least likely a purpose of the auditors rapid product obsolescence.
inquiries 21. Which of the following is most likely an example of fraud risk factor relating to managements
of management? characteristics and influence over the control environment?
a. To obtain an understanding of managements assessment of the risk that the financial a. There is a strained relationship between management and the current or predecessor
statements may be materially misstated as a result of fraud. auditor.
b. To obtain knowledge of managements understanding regarding the accounting and internal b. Inability to generate cash flows from operations while reporting earnings and earnings
control systems in place to prevent and detect error. growth.
c. To determine whether management has discovered any material errors. c. Significant related party transactions which are not in the ordinary course of business.
d. To determine extent of authentication of documentation. d. Significant, unusual or highly complex transactions (especially those close to year-end) that
16. Which of the following best describes what is meant by the term fraud risk factor? pose difficult questions concerning substance over form.
a. Factors whose presence indicates that the risk of fraud is high. 22. Examples of fraud risk factors relating to susceptibility of assets to misappropriation include
the irregularities.
following, except c. He should consider the potential effect on the financial statements.
a. Large amounts of cash on hand or processed. d. He should refer the suspected fraud or error to the internal auditor.
b. Inventory characteristics, such as small size combined with high value and high demand. 28. If the auditor believes an indicated fraud or error could have a material effect on the financial
c. Easily convertible assets, such as bearer bonds, diamonds or computer chips. statements, the nature, timing and extent of the procedures to be performed depends on the
d. Lack of appropriate management oversight. auditors judgment as to
23. Judgments about the risk of material misstatements resulting from fraud may affect the audit a. The type of fraud or error.
in b. The likelihood that a particular type of fraud or error could have a material effect on the
the following ways, except financial statements.
a. The application of professional skepticism may include increased sensitivity in the selection c. The likelihood of their occurrence.
of the nature and extent of documentation to be examined in support of material d. All of the above.
transactions. 29. The auditor should document
b. The knowledge, skill and ability of members of the audit team assigned significant audit a. Fraud risk factors identified as being present during the auditors assessment process.
responsibilities need to be commensurate with the auditors assessment of the level of risk b. The auditors response to fraud risk factors identified.
for the engagement. c. Both a and b.
c. The auditor may decide to consider further managements selection and application of d. Neither a nor b.
significant accounting policies, particularly those related to revenue recognition, asset 30. The auditor least likely obtains written representations from management that the
valuation or capitalizing versus expensing. management:
d. The auditors ability to assess control risk at high level may be reduced. a. Acknowledges its responsibility for the implementation and operations of accounting and
24. The nature, timing and extent of procedures may need to be modified in the following ways internal control systems that are designed to prevent and detect fraud and error.
as b. Believes the effects of those uncorrected financial statement misstatements aggregated by
possible responses to the auditors assessment of the risk of material misstatement resulting the auditor during the audit are material, both individually and in the aggregate, to the
from both fraudulent financial reporting and misappropriation of assets. financial statements taken as a whole.
a. The nature of audit procedures performed may need to be changed to obtain evidence that c. Has disclosed to the auditor all significant facts relating to any frauds or suspected frauds
is more reliable or to obtain additional corroborative information. known to management that may have affected the entity.
b. The timing of substantive procedures may need to be altered to be closer to, or at, year-end. d. Has disclosed to the auditor the results of its assessment of the risk that the financial
c. The extent of the procedures applied will need to reflect the assessment of the risk of statements may be materially misstated as a result of fraud.
material misstatement resulting from fraud. 31. Communication of a misstatement resulting from fraud, or a suspected fraud, or error to the
d. All of the above. appropriate level of management on a timely basis is important because it enables management
25. The auditor may encounter circumstances that, individually or in combination, indicate the to take action as necessary. Ordinarily, the appropriate level of management is
possibility that the financial statements may contain a material misstatement resulting from fraud a. At least equal to the level of the persons who appear to be involved with the misstatement or
or error. These circumstances include the following, except suspected fraud.
a. Unrealistic time deadlines for audit completion imposed by management. b. At least one level above the persons who appear to be involved with the misstatement or
b. Conflicting or unsatisfactory evidence provided by management or employees. suspected fraud.
c. Information provided unwillingly or after unreasonable delay. c. The audit committee of the board of directors.
d. Transactions recorded in accordance with managements general or specific authorization. d. The head of internal audit department.
26. Which of the following circumstances most likely indicate the possibility of fraud or error? 32. The auditor may encounter exceptional circumstances that bring into question the auditors
a. Management engages in frank communication with appropriate third parties, such as ability to continue performing the audit, including where
regulators and bankers. a. The entity does not take the remedial action regarding fraud that the auditor considers
b. Evidence of an unduly lavish lifestyle by officers or employees. necessary in the circumstances, even when the fraud is not material to the financial
c. Conservative application of accounting principles. statements.
d. Minimal differences from expectations disclosed by analytical procedures. b. The auditors consideration of the risk of material misstatement resulting from fraud and the
27. Which of the following should the auditor likely to do when the application of planned audit results of audit tests indicate a significant risk of material and pervasive fraud.
procedures indicates the possible existence of fraud or error? c. The auditor has significant concern about the competence or integrity of management or
a. The auditor should resign in order to avoid legal responsibility. those charged with governance.
b. He should discuss the matter with the person whom he believes is involved with the d. All of the above.
PSA 250 Consideration of Laws and Regulations in an Audit of Financial Statements to act, the auditor withdrew from the engagement. The auditors decision to withdraw was
1. When an auditor becomes aware of a possible illegal act by a client, the auditor should obtain primarily due to doubt concerning
an a. Inadequate financial statement disclosures.
understanding of the nature of the act to b. Compliance with the laws.
a. Increase the assessed level of control risk. c. Scope limitations resulting from the inaction.
b. Recommend remedial actions to the audit committee. d. Reliance on managements representation.
c. Evaluate the effect on the financial statements. 7. Which of the following is incorrect about the auditors responsibility of evaluating
d. Determine the reliability of managements representations. noncompliance
2. Mac, CPA, is auditing the financial statements of TLs Retailing, Inc. What assurance does by the entity to laws and regulations?
Mac a. An audit cannot be expected to detect noncompliance with all laws and regulations.
provide that direct effect illegal acts that are material to TLs financial statements, and illegal b. Noncompliance refers to acts of omission or commission by the entity being audited which
acts are contrary to prevailing laws or regulations.
that have a material, indirect effect on the financial statements will be detected? c. Noncompliance includes personal misconduct of entity management or employers though
Direct effect illegal acts Indirect effect illegal acts they are unrelated to the entitys business activities.
a. Reasonable None d. Detection of noncompliance, regardless of materiality, requires considerations of the
b. Reasonable Reasonable implications for the integrity of management or employees.
c. Limited None 8. What is expected of auditor in determining noncompliance by an entity to existing laws and
d. Limited Reasonable regulations?
3. The most likely explanation why the auditors examination cannot reasonably be expected to a. Whether an act constitutes noncompliance is a legal determination that is ordinarily within
bring all illegal acts by the client to the auditors attention is that the auditors professional competence.
a. Illegal acts are perpetrated by management override of internal accounting controls. b. The auditors training, experience and understanding of the entity and its industry cannot
b. Illegal acts by clients often relate to operating aspects rather than accounting aspects. provide a basis for recognition that some acts coming to the auditors attention may
c. The clients system of internal accounting control may be so strong that the auditor performs constitute noncompliance with laws and regulations.
only minimal substantive testing. c. The determination as to whether a particular act constitutes or is likely to constitute
d. Illegal acts may be perpetrated by the only person in the clients organization with access to noncompliance is generally based on the understanding of the auditor but ultimately can only
both assets and the accounting records. be determined by an expert who is qualified to practice law.
4. An auditor who finds that the client has committed an illegal act would be most likely to d. In order to plan the audit, the auditor should obtain a general understanding of the legal and
withdraw regulatory framework applicable to the entity and the industry and how the entity is
from the engagement when the complying with the framework.
a. Illegal act affects auditors ability to rely on management representations. 9. When the auditor becomes aware of information concerning a possible noncompliance to
b. Illegal act has material financial statement implications. laws
c. Illegal act has received widespread publicity. or regulations, the auditor should appropriately:
d. Auditor cannot reasonably estimate the effect of the illegal act on the financial statements. a. Obtain an understanding of the nature of the act and the circumstances in which it has
5. If an auditor believes a client may have committed illegal acts, which of the following actions occurred, and evaluate the possible effect on the financial statements.
should the auditor take? b. Discuss his suspicion with the management.
a. Consult with the clients counsel and the auditors counsel to determine how the suspected c. Ask management to determine whether a violation is really committed.
illegal acts will be communicated to stockholders. d. Consult with the entitys legal counsel as to what appropriate action the auditor should do.
b. Extend auditing procedures to determine whether the suspected illegal acts have a material 10. If the auditor suspects that members of senior management, including members of the
effect on the financial statements. board of
c. Make inquiries of the clients management and obtain an understanding of the directors, are involved in noncompliance to laws as regulations, and he believes his report may
circumstances underlying the acts and of other evidence to determine the effects of the acts not be acted upon, he would:
on the financial statements. a. Do nothing.
d. Notify each member of the audit committee of the board of directors about the nature of the b. Issue a disclaimer of opinion.
acts and request that they advise an approach to be taken by the auditor. c. Consider seeking legal advice.
6. An audit clients board of directors and audit committee refused to take action about an d. Make special investigation in order to fully determine the extent of clients noncompliance.
immaterial illegal act that was brought to their attention by the auditor. Because of their failure 11. Which of the following circumstances regarding the entitys noncompliance to laws or
regulations c. The auditor is required, in an audit in accordance with PSAs, to design procedures for the
may cause the auditor to resign from an engagement? specific purpose of identifying these matters.
a. The auditor is unable to determine whether noncompliance has occurred. d. The auditor is not required to communicate these matters with those charged with
b. If the auditor concludes that the noncompliance has a material effect on the financial governance of an entity.
statements and has not been properly reflected in the financial statements. 5. Which statement is incorrect regarding the auditors communications of audit matters with
c. When the entity does not take remedial action that he considers necessary in the those
circumstances even when the noncompliance is not material to financial statements. charged with governance?
d. When the disclosure of the effect of noncompliance to legal authority is necessary. a. The auditor should communicate audit matters of governance interest upon completion of
12. Examples of the type of information that may come to the auditor's attention that may the engagement.
indicate b. The auditors communications with those charged with governance may be made orally or in
that noncompliance with laws or regulations has occurred least likely include writing.
a. Investigation by government departments or payment of fines or penalties. c. When audit matters of governance interest are communicated orally, the auditor documents
b. Sales commissions or agent's fees that appear reasonable in relation to those ordinarily paid in the working papers the matters communicated and any responses to those matters.
by the entity or in its industry or to the services actually received. d. Ordinarily, the auditor initially discusses audit matters of governance interest with
c. Unusual transactions with companies registered in tax havens. management, except where those matters relate to questions of management competence
d. Media comment. or integrity.
PSA 260 Communications of Audit Matters with Those Charged with Governance Other Professional Responsibilities
1. Which statement is incorrect regarding PSA 260? 1. An auditors overall objective in a financial statement audit is to
a. The purpose of this PSA is to establish standards and provide guidance on communication a. Determine that all individual accounts and footnotes are fairly presented.
of audit matters arising from the audit of financial statements between the auditor and those b. Employ the audit risk model.
charged with governance of an entity. c. Express an opinion on the fair presentation of the financial statements in accordance with
b. These communications relate to audit matters of governance interest as defined in this PSA. generally accepted accounting principles.
c. This PSA provides guidance on communications by the auditor to parties outside the entity, d. Detect all errors and fraud.
for example, external regulatory or supervisory agencies. 2. The primary responsibility for the adequacy of disclosure in the financial statements of a
d. All the above statements are correct. publicly
2. Which statement is incorrect regarding the auditors communications of audit matters with held company rests with the
those a. Partner assigned to the audit engagement. c. Auditor in-charge of field work.
charged with governance? b. Management of the company. d. Securities and Exchange Commission.
a. The auditor should communicate audit matters of governance interest arising from the audit 3. Reasonable assurance means:
of financial statements with those charged with governance of an entity. a. Gathering of all available corroborating evidence for the auditor to conclude that there are no
b. Those charged with governance ordinarily are accountable for ensuring that the entity material misstatements in the financial statements, taken as a whole.
achieves its objectives, financial reporting, and reporting to interested parties. b. Gathering of the audit evidence necessary for the auditor to conclude that there are no
c. Audit matters of governance interest are those that arise from the audit of financial material misstatements in the financial statements, taken as a whole.
statements and, in the opinion of the auditor, are either important or relevant to those c. Gathering of the audit evidence necessary for the auditor to conclude that the financial
charged with governance in overseeing the financial reporting and disclosure process. statements, taken as a whole, are free from any misstatements.
d. Audit matters of governance interest include only those matters that have come to the d. Gathering of the audit evidence necessary for the auditor to conclude that the financial
attention of the auditor as a result of the performance of the audit. statements are free of material unintentional misstatements.
3. The role of persons entrusted with the supervision, control and direction of an entity 4. Which of the following ultimately determines the specific audit procedures necessary to
a. Governance c. Government provide
b. Board of directors d. Management an independent auditor with a reasonable basis for the expression of an opinion?
4. Which statement is correct regarding audit matters of governance interest? a. the audit program. c. generally accepted auditing standards.
a. These are matters that arise from the audit of financial statements and, in the opinion of the b. the auditors judgment. d. the auditors working papers.
auditor, are either important or relevant to those charged with governance in overseeing the 5. Which of the following best describes a trend in litigation involving CPAs?
financial reporting and disclosure process. a. A CPA cannot render an opinion on a company unless the CPA has audited all affiliates of
b. These include only those matters that have come to the attention of the auditor as a result of that company.
the performance of the audit. b. A CPA may successfully assert as a defense that the CPA had no motive to be part of a
fraud. b. The treasurer was Glass' agent and, therefore, Glass was responsible for preventing the
c. A CPA may be exposed to criminal as well as civil liability. embezzlement.
d. A CPA is primarily responsible for a clients footnotes in an annual report filed with the SEC. c. The financial statements were presented in conformity with GAAP.
6. In performing MAS engagements, CPAs should not take any positions that might d. Mix had no actual knowledge of the embezzlement.
a. Constitute advice and assistance 5. The factor that distinguishes constructive fraud from actual fraud is
b. Provide technical assistance in implementation a. Materiality c. Quality of internal control.
c. Result in new organizational policies and procedures b. Type of error or irregularity d. Intent.
d. Impair their objectivity 6. Working papers prepared by a CPA in connection with an audit engagement are owned by
7. An audit independence issue might be raised by the auditors participation in management the
advisory services engagements. Which of the following statements is most consistent with the CPA, subject to certain limitations. The rationale for this rule is to
professions attitude toward this issue? a. Protect the working papers from being subpoenaed.
a. Information obtained as a result of a management advisory services engagement is b. Provide the basis for excluding admission of the working papers as evidence because of the
confidential to that specific engagement and should not influence performance of the attest privileged communication rule.
function. c. Provide the CPA with evidence and documentation which may be helpful in the event of a
b. The decision as to loss of independence must be made by the client based upon the facts of lawsuit.
the particular case. d. Establish a continuity of relationship with the client whereby indiscriminate replacement of
c. The auditor should not make management decisions for an audit client. CPAs is discouraged.
d. The auditor who is asked to review management decisions is also competent to make these 7. Mead Corp. orally engaged Dex & Co., CPAs, to audit its financial statements. The
decisions and can do so without loss of independence. management of Mead informed Dex that it suspected that the accounts receivable were
8. The form of communication with a client in a management advisory service consultation materially overstated. Although the financial statements audited by Dex did, in fact, include a
should materially overstated accounts receivable balance, Dex issued an unqualified opinion. Mead
be relied on the financial statements in deciding to obtain a loan from City Bank to expand its
a. Either oral or written. operations. City relied on the financial statements in making the loan to Mead. As a result of
b. Oral with appropriate documentation in the work papers. the overstated accounts receivable balance, Mead has defaulted on the loan and has incurred a
c. Written and copies should be sent to both management and the board of directors. substantial loss. If Mead sues Dex for negligence in failing to discover the overstatement, Dex's
d. Written and a copy should be sent to management alone. best defense would be that
Legal Responsibilities a. No engagement letter had been signed by Dex.
1. Which one of the following, if present, would support a finding of constructive fraud on the b. The audit was performed by Dex in accordance with generally accepted auditing standards.
part of c. Dex was not in privity of contract with Mead.
a CPA? d. Dex did not perform the audit recklessly or with an intent to deceive.
a. Privity of contract. c. Intent to deceive. 8. As a consequence of failure to adhere to generally accepted auditing standards in the course
b. Reckless disregard. d. Ordinary negligence. of
2. The limitation of auditor liability under contract law is known as an audit of the Lamp Corp., Harrison, CPA, did not detect the embezzlement of a material
a. Privity of contract. c. Contributory liability. amount of funds by the company's controller. As a matter of common law, to what extent would
b. Statutory liability. d. Common law liability. Harrison be liable to the Lamp Corp. for losses attributable to the theft?
3. The auditor's defense of contributory negligence is most likely to prevail when a. No liability since the ordinary examination cannot be relied on to detect defalcations.
a. Third party injury has been minimal. b. No liability because privity of contract is lacking.
b. The auditor fails to detect fraud resulting from management override of the control structure. c. Liable for losses attributable to her or his negligence.
c. The client is privately held as contrasted with a public company. d. Liable only if it could be proved that he or she was grossly negligent.
d. Undetected errors have resulted in materially misleading financial statements. 9. Martin Corporation orally engaged Humm & Dawson to audit its year-end financial
4. Mix and Associates, CPAs, issued an unqualified opinion on the financial statements of Glass statements.
Corp. for the year ended December 31, 2005. It was determined later that Glass' treasurer had The engagement was to be completed within two months after the close of Martin's fiscal year
embezzled P3,000,000 from Glass during 2005. Glass sued Mix because of Mix's failure to for a fixed fee of P250,000. Under these circumstances, what obligation is assumed by Humm
discover the embezzlement. Mix was unaware of the embezzlement. Which of the following is & Dawson?
Mix's best defense? a. None. The contract is unenforceable since it is not in writing.
a. The audit was performed in accordance with GAAS. b. An implied promise to exercise reasonable standards of competence and care.
c. An implied obligation to take extraordinary steps to discover all defalcations. retain a client, the firms independence and ability to serve the client properly and the
d. The obligation of an insurer of its work, which is liable without fault. integrity of the clients management are to be considered.
10. In which of the following statements about a public accounting firm's action is scienter or its 7. Monitoring: The continued adequacy and operational effectiveness of quality control
equivalent absent? policies and procedures is to be monitored.
a. Reckless disregard for the truth. Individual Audit Level: The auditor should implement those quality control procedures which
b. Actual knowledge of fraud. are,
c. Intent to gain monetarily by concealing fraud. in the context of the policies and procedures of the firm, appropriate to the individual audit.
d. Performance of substandard auditing procedures. 1. Direction: Direction involves informing assistants of their responsibilities and the objectives
11. The leading precedent-setting auditing case in the third party liability is of the procedures they are to perform. It also involves informing them of matters, such as
a. Escott et al. v. Bar Chris Construction Corp. the nature of the entitys business and possible accounting or auditing problems that may
b. Hochfelder v. Ernst & Ernst. affect the nature, timing and extent of audit procedures with which they are involved.
c. Ultramares Corporation v. Touche. 2. Supervision: Supervision is closely related to both direction and review and may involve
d. United States v. Simon. elements of both. Personnel carrying out supervisory responsibilities perform the following
12. The leading case of criminal action against CPAs is the functions during the audit:
a. 1136 Tenants case. a. monitor the progress of the audit to consider whether (1) assistants have the
b. United States v. Simon case, aka Continental Vending. necessary skills and competence to carry out their assigned tasks; (2) assistants
c. Escott et al. v. Bar Chris case, aka Bar Chris. understand the audit directions; and (3) the work is being carried out in accordance
d. Ultramares Corporation v. Touche case with the overall audit plan and the audit program.
b. become informed of and address significant accounting and auditing questions raised
during the audit
CPA REVIEW SCHOOL OF THE PHILIPPINES c. resolve any differences of professional judgment between personnel and consider the
Manila level of consultation that is appropriate.
AUDITING THEORY 3. Review: The work performed by each assistant needs to be reviewed by personnel of least
QUALITY CONTROLS AND GENERALLY ACCEPTED AUDITING equal competence.
STANDARDS (GAAS) GENERALLY ACCEPTED AUDITING STANDARDS
Related PSA : PSA 220 General standards
QUALITY CONTROL FOR AUDIT WORK 1. The examination is to be performed by a person or persons having adequate technical
training
Audit Firm Level: The audit firm should implement quality control policies and
and proficiency as an auditor.
procedures 2. In all matters relating to the assignment, an independence in mental attitude is to be
designed to ensure that all audits are conducted in accordance with PSAs or relevant maintained by the auditor or auditors.
national 3. Due professional care is to be exercised in the performance of the examination and the
standards or practices. preparation of the report.
1. Professional requirements: independence, integrity, objectivity, confidentiality and Standards of Fieldwork
professional behavior. 1. The work is to be adequately planned, and assistants, if any, are to be properly supervised.
2. Skills and competence: The firm is to be staffed by personnel who have attained and 2. There is to be a proper study and evaluation of the existing internal control as a basis for
maintained the technical standards and professional competence required to enable them reliance thereon and for the determination of the resultant extent of the tests to which auditing
to fulfill their responsibilities with due care. procedures are to be restricted.
3. Assignment: Audit work is to be assigned to personnel who have the degree of technical 3. Sufficient, competent evidential matter is to be obtained through inspection, observation,
training and proficiency required in the circumstances. inquiries, and confirmations to afford a reasonable basis for an opinion regarding the financial
4. Delegation: There is to be sufficient direction, supervision and review of work at all levels to statements under examination.
provide reasonable assurance that the work performed meets appropriate standards of Standards of Reporting
quality. 1. The report shall state whether the financial statements are presented in accordance with
5. Consultation: Whenever necessary, consultation within or outside the firm is to occur with generally accepted principles of accounting.
those who have appropriate expertise. 2. The report shall identify those circumstances in which principles have not been consistently
6. Acceptance and retention of clients: An evaluation of prospective clients and a review, on observed in the current period in relation to the preceding period.
an ongoing basis, of existing clients is to be conducted. In making a decision to accept or
3. Informative disclosures are to be regarded as reasonably adequate unless otherwise stated in b. Professional education that is required in order to perform with due professional care.
the report. c. Knowledge required to fulfill assigned responsibilities and to progress within the firm.
4. The report shall either contain an expression of opinion regarding the financial statements, d. Knowledge required in order to perform a peer review.
taken as a whole, or an assertion to the effect that an opinion cannot be expressed. When an 8. In pursuing its quality control objectives with respect to assigning personnel to engagements,
overall opinion cannot be expressed, the reasons therefor should be stated. In all cases where a public accounting firm may use policies and procedures such as
an auditors name is associated with financial statements, the report should contain a clear-cut a. Rotating employees from assignment to assignment on a random basis to aid in the staff
indication of the character of the auditors examination, if any, and the degree of responsibility training effort.
the auditor is taking. b. Requiring timely identification of the staffing requirements of specific engagements so that
MULTIPLE CHOICE QUESTIONS enough qualified personnel can be made available.
1. A basic objective of a CPA firm is to provide professional services that conform with c. Allowing staff to select the assignments of their choice to promote better client
professional standards. Reasonable assurance of achieving this basic objective is provided relationships.
through d. Assigning a number of employees to each engagement in excess of the number required
a. A system of peer review. so as not to overburden the staff and interfere with the quality of the audit work
b. Continuing professional education. performed.
c. A system of quality controls. 9. A CPA firms personnel partner periodically studies the CPA firms personnel advancement
d. Compliance with generally accepted reporting standards. experience to ascertain whether individuals meeting stated criteria are assigned increased
2. The examination by CPAs of a CPA firms auditing practices to ascertain compliance with its degrees of responsibility. This is evidence of the CPA firms adherence to prescribed
quality control system standards of
a. Compliance audit c. Peer review a. Quality control. c. Supervision and review.
b. Examination d. Quality control audit b. Due professional care. d. Fieldwork.
3. Quality control policies and procedures are required to be implemented at 10. The firms evaluation of the performance of its personnel and advising them of their progress
abcd is a quality control procedure that relates to
Audit firm level Yes Yes No No a. Promotion c. Monitoring
Individual audit level Yes No Yes No b. Advancement d. Directing
4. The following factors affect the nature, timing and extent of an audit firms quality control 11. Which of the following relate to skills and competence as an objective of quality control
policies and procedures, except policies?
abcd a. Advancement c. Professional development
Size and nature of practice Yes Yes No No b. Hiring d. All of these
Geographic dispersion Yes Yes Yes No 12. Which of the following practices will promote the objectives of assignment of personnel?
Organization Yes No Yes No A. Evaluates partners periodically by means of senior partner or fellow partner evaluation
Appropriate cost/benefit considerations Yes Yes No No and counseling as to whether they continue to have the qualifications to fulfill their
5. The firm is to be staffed by personnel who have attained and maintained the technical responsibilities.
standards and professional competence required to enable them to fulfill their responsibilities B. Identifies on a timely basis the staffing requirements of specific audits
with due care is the objective of what quality control policy? C. Periodically counsels personnel as to their progress and career opportunities
a. Professional Requirements c. Assignment D. Prepares time budget for audit to determine manpower requirements and to schedule
b. Skills and Competence d. Delegation audit work.
6. In connection with the element of professional development, a CPA firms system of quality a. A b. A C c. B D d. All of them
control should ordinarily provide that all personnel 13. Which of the following is not likely a quality control procedure on consultation?
a. Have the knowledge required to enable them to fulfill responsibilities assigned. a. Identifies areas and specialized situations where consultation is required and encourage
b. Possess judgment, motivation, and adequate experience. personnel to consult with or in use authoritative sources on other complex matters.
c. Seek assistance from persons having appropriate levels of knowledge, judgment, and b. Designates individuals as specialists to serve as authoritative sources and define their
authority. authority in consultative situations.
d. Demonstrate compliance with peer review directives. c. Assigns an appropriate person or persons to be responsible for assigning personnel to
7. Within the context of quality control, the primary purpose of continuing professional education audits.
and training activities, is to enable a CPA firm to provide personnel within the firm with: d. Specifies the extent of documentation to be provided for the result of consultation in those
a. Technical training that assures proficiency as an auditor. areas and specialized situations where consultation is required.
14. Monitoring, as an element of quality control policies of a firm, requires: b. No No Yes
a. Providing reasonable assurance that the firms other quality control policies and c. Yes No Yes
procedures are effectively operating. d. yes Yes No
b. Designates individuals as specialists to serve as authoritative sources and define their 22. Which one of the following relates to delegation objective of quality control?
authority in consultative situations. a. The firms creates a group that provides technical training to audit staff.
c. Ensuring that personnel are sufficiently directed, supervised and their work being b. There is to be a sufficient direction, supervision, and review of work at all levels to provide
reviewed adequately. reasonable assurance that the work performed meets appropriate standards of quality.
d. Identify the right personnel to be assigned in an audit engagement. c. Assignment of work to the more qualified personnel.
15. Which of the following quality control procedures is a monitoring activity? d. Whenever necessary, consultation within or outside the firm is to occur with those who
a. Evaluates the firms independence and its ability to serve the prospective client have appropriate expertise.
b. Reviews and tests compliance with the firms general quality control policies and 23. Which of the following is (are) helpful in communicating audit directions?
procedures. abcd
c. Designates individuals as specialists to serve as authoritative sources and define their Audit program Yes Yes Yes No
authority in consultative situations. Overall audit plan Yes Yes No No
d. Monitors continuing professional education programs and maintain appropriate records, Time budgets Yes No No No
both on a firm and an individual audit engagement basis. 24. Generally Accepted Auditing Standards (GAAS) and Philippine Standards on Auditing (PSA)
16. Which of the following objectives are generally a component of a firms quality control? should be looked upon by practitioners as:
A. Professional requirements E. Consultation a. Ideals to work towards, but which are not achievable
B. Skills and competence F. Due professional care b. Maximum standards which denote excellent work.
C. Assignment G. Monitoring c. Minimum standards of performance which must be achieved on each audit engagement.
D. Inspection H. Delegation d. Benchmark to be used on all audits, reviews, and compilations.
a. A, B, C, D, E, F c. A, B, C, E, G, H 25. Which of the following best describes what is meant by Generally Accepted Auditing
b. A, B, C, F, E, G d. B, C, G, F, H Standards?
17. Which of the following is not an element of professional requirements as prescribed by a. Pronouncements issued by the Auditing Standards and Practices Council.
Quality Control Policies for an audit firm? b. Procedure to be used to gather evidence to support financial statements.
a. Independence c. Confidentiality c. Rules acknowledged by the accounting profession because of their universal compliance.
b. Integrity d. Prudence d. Measures of the quality of the auditors performance
18. Which of the following is an element of directing an audit assistant objective? 26. An auditor need not abide by an auditing standard if the auditor believes that
a. Identifying in advance the staffing requirements of a particular audit engagement. a. The amount is immaterial
b. Informing assistants of their responsibilities and the objectives of the procedures they are b. The requirement of the standard is impractical to perform
to perform. c. The requirement of the standard is impossible to perform
c. Resolving any differences in professional judgment between audit personnel. d. Any of the three above is correct.
d. Resolution of differences in audit findings. 27. A CPA should comply with applicable generally accepted auditing standards on every
19. It involves informing assistants of their responsibilities and the objectives of the procedures engagement
they have to perform: a. Without exception
a. Supervision b. Monitoring c. Directing d. Consultation b. Except in examinations that result in a qualified report
20. What is the overriding reason why the auditor considers the professional competence of c. Except in engagements where the CPA is associated with unaudited financial statements.
assistants whom the work will be delegated? d. Except in examinations of interim financial statements.
a. All the audit assistants assigned to an engagement must be independent in appearance. 28. To exercise due professional care the auditor should examine all available corroborating
b. To have reasonable assurance that such work will be performed with due care by the evidences supporting managements assertions.
audit assistant. The proper attitude of an auditor who is performing an examination in accordance with GAAS
c. To lessen the working paper preparation. should be professional responsiveness.
d. To eliminate audit risk. GAAS means rules acknowledged by the accounting profession because of their universal
21. Which of the following is(are) quality control policies on an audit firm level? application.
Consultation Assignment Direction a. b. c. d.
a. Yes No Yes First statement True False True True
Second statement False False True False b. Obtain a knowledge of matters that relate to the nature of the entity's business.
Third statement False False True True c. Refer a substantial portion of the audit to another CPA who will act as the principal
29. A CPA is most likely to refer to one or more of the three general auditing standards in auditor.
determining d. First inform management that an unqualified opinion cannot be issued.
a. The nature of the CPAs auditing qualification. 37. In any case in which the CPA or the CPAs assistants are not qualified to perform the work,
b. The scope of the CPAs auditing procedures. a
c. Requirements for the review of internal control. professional obligation exists to
d. Whether the CPA should undertake an audit engagement. a. Acquire the requisite knowledge and skills
30. The general standards stress the importance of b. Suggest someone else who is qualified to perform the work
a. The personal qualities which the auditor should have c. Decline the engagement
b. Evidence accumulation d. Any of the above
c. Communicating the auditors finding to the reader 38. A CPA, while performing an audit, strives to achieve independence in appearance in order
d. All of the above to
31. The Audit Standard which requires adequate technical training and proficiency is normally a. Reduce risk and liability.
interpreted as requiring the auditor to have b. Comply with the generally accepted standards of field work
a. Formal education in auditing and accounting c. Become independent in fact.
b. Adequate practical experience for the work being performed d. Maintain public confidence in the profession.
c. Continuing professional education 39. The standard of due audit care requires the auditor to
d. All of the above a. Apply judgment in a conscientious manner, carefully weighing the relevant factors before
32. Which of the following is not required by the Generally Accepted Auditing Standards that reaching a decision.
states that due professional care is to be exercised in the performance of the audit? b. Ensure that the financial statements are free from error.
a. Observance of the standards of field work and reporting c. Make perfect judgment decisions in all cases.
b. Critical review of the audit work performed at every level of supervision d. Possess skills clearly above the average for the profession.
c. Degree of skill commonly possessed by others in the profession. 40. The third general standards states that due care is to be exercised in the performance of the
d. Responsibility for losses because of errors of judgment examination. This standard should be interpreted to mean that a CPA who undertakes an
33. The first general standard requires that the audit of financial statements be performed by a engagement assumes a duty to perform.
person or persons having adequate technical training and a. With reasonable diligence and without fault or error.
a. Independence with respect to the financial statements and supplementary disclosures. b. As a professional who will assume responsibility for losses consequent upon error of
b. Exercising professional care as judged by peer reviewers. judgment.
c. Proficiency as an auditor, which likely has been acquired from previous experience. c. To the satisfaction of the client and third parties who may rely upon it.
d. Objectivity as an auditor, as verified by proper supervision. d. As a professional possessing the degree of skill commonly possessed by others in the
34. Which of the following is mandatory if the auditor is to comply with generally accepted field.
auditing standards? 41. The third general standard states due care is to be exercised in the performance of an audit.
a. Possession by the auditor of adequate technical training. This standard is generally interpreted to require
b. Use of analytical review on audit engagements. a. Objective review of the adequacy of the technical training and proficiency of firm
c. Use of statistical sampling whenever feasible on an audit engagement. personnel
d. Confirmation by the auditor of material accounts receivable balances. b. Critical review of work done at every level of supervision
35. Competence as a certified public accountant includes all of the following except c. Thorough review of the existing internal control structure
a. Having the technical qualifications to perform an engagement. d. Periodic review of a CPA firms quality control procedures.
b. Possessing the ability to supervise and evaluate the quality of staff work. 42. The first standard of fieldwork, which states that the work is to be adequately planned, and
c. Warranting the infallibility of the work performed. assistants, if any, are to be properly supervised, recognizes that
d. Consulting others if additional technical information is needed. a. Early appointment of the auditor is advantageous both to the auditor and to the client.
36. An auditor who accepts an audit engagement and does not possess the industry expertise b. Acceptance of an audit engagement after the close of the client's fiscal year is generally
of not permissible.
the business entity, should c. Appointment of the auditor subsequent to the physical count of inventories requires a
a. Engage financial experts familiar with the nature of the business entity. disclaimer of opinion.
d. Performance of substantial parts of the engagement is necessary at interim dates.
43. With respect to the auditors planning of a year-end audit, which of the following statements
is
always true?
a. An engagement should not be accepted after the fiscal year-end.
b. An inventory count must be observed at the balance sheet date.
c. The clients audit committee should not be told of the specific audit procedures that were
performed.
d. It is an acceptable practice to carry out part of the audit at interim dates.
44. Which of the following statements is incorrect?
a. The auditors report must state whether the financial statements were prepared with
GAAP.
b. The auditors report must state whether GAAP was consistently followed from the prior
period to the current period.
c. The auditors report must imply whether the client has provided adequate disclosure on
the financial statements and in the accompanying notes.
d. The auditors report must express an opinion on the financial statements taken as a
whole, or explain why there is no opinion provided.

You might also like